Implicit Differentiation: Finding dy/dx for cos(y^2) = x^4

Click For Summary
To find dy/dx for the equation cos(y^2) = x^4, the implicit differentiation process involves differentiating both sides, leading to -sin(y^2) * 2y * dy/dx = 4x^3. The correct expression for dy/dx is -4x^3/(2y sin(y^2)), which simplifies to -2x^3/(y sin(y^2)). Among the multiple-choice answers provided, option B is identified as correct, while the other options are deemed incorrect. The discussion emphasizes the importance of careful notation and understanding in implicit differentiation.
mlowery
Messages
23
Reaction score
0
Find \frac{dy}{dx} given cos(y^2) = x^4
Is this correct:

1. cos(y^2) = x^4

2. -sin(y^2) \times 2y \frac{dy}{dx} = 4x^3

3. \frac{dy}{dx} = \frac{4x^3}{-2sin(y^2)}
 
Last edited:
Physics news on Phys.org
Oops... Looks like you left out a "y" in the last step. Write it like this:
\frac{dy}{dx} = -\frac{2x^3}{y \sin(y^2)}
 
Last edited:
Yes, that is what I originally did. The thing is, this is a multiple choice question. Of the choices, the only answers close to this are: (Please don't think I am using this forum for answers. I just believe none of the choices are correct).

A) \frac{4x^3}{-sin(y^2)}

B) \frac{4x^3}{-2ysin(y^2)}

Here are the other choices:

C) \frac{\sqrt{xy}-y}{2xy}

D) \frac{x^4}{-sin(y^2)}

E) \frac{4x^3}{cos(2y)}
 
For the second step, I'd prefer having it written as:

-sin(y^2) \times 2y dy = 4x^3dx

This will probably be more consistent once you encounter more complicated problems or do multivariable calculus.

Answer B is correct. You forgot to move the y over in your last step, step 3.
 
Last edited:
mezarashi said:
Answer B is correct. You forgot to move the y over in your last step, step 3.
Right! (I just realized that you left out that y in your last step!)
 
Hehe, how'd that "y" sneak by me?
Thanks for the help.

Yeah mezarashi, I am not quite familiar with the notation you used (I just started using the dy/dx notation last week).

Thanks,
Mitch
 
Question: A clock's minute hand has length 4 and its hour hand has length 3. What is the distance between the tips at the moment when it is increasing most rapidly?(Putnam Exam Question) Answer: Making assumption that both the hands moves at constant angular velocities, the answer is ## \sqrt{7} .## But don't you think this assumption is somewhat doubtful and wrong?

Similar threads

  • · Replies 2 ·
Replies
2
Views
2K
Replies
3
Views
2K
  • · Replies 15 ·
Replies
15
Views
2K
Replies
4
Views
2K
  • · Replies 8 ·
Replies
8
Views
1K
  • · Replies 5 ·
Replies
5
Views
1K
  • · Replies 3 ·
Replies
3
Views
2K
Replies
6
Views
2K
Replies
3
Views
2K
  • · Replies 24 ·
Replies
24
Views
2K